Difference between revisions of "2016 AMC 12A Problems/Problem 3"

(Solution)
(Solution)
Line 1: Line 1:
 
==Solution==
 
==Solution==
 
<cmath>\text{rem}\left(\frac{3}{8},-\frac{2}{5}\right)</cmath>
 
<cmath>\text{rem}\left(\frac{3}{8},-\frac{2}{5}\right)</cmath>
<cmath>=\frac{3}{8}-\left(-\frac{2}{5}\right)\lfloor\frac{\frac{3}{8}}{-\frac{-2}{5}}\rfloor</cmath>
+
<cmath>=\frac{3}{8}-\left(-\frac{2}{5}\right)\lfloor\frac{\frac{3}{8}}{-\frac{2}{5}}\rfloor</cmath>
 
<cmath>=\frac{3}{8}+\left(\frac{2}{5}\right)\lfloor -\frac{15}{16}\rfloor</cmath>
 
<cmath>=\frac{3}{8}+\left(\frac{2}{5}\right)\lfloor -\frac{15}{16}\rfloor</cmath>
 
<cmath>=\frac{3}{8}+\left(\frac{2}{5}\right)\left(-1\right)</cmath>
 
<cmath>=\frac{3}{8}+\left(\frac{2}{5}\right)\left(-1\right)</cmath>
 
<cmath>=\frac{3}{8}-\frac{2}{5}</cmath>
 
<cmath>=\frac{3}{8}-\frac{2}{5}</cmath>
 
<cmath>=\boxed{\textbf{(B)}-\frac{1}{40}}</cmath>
 
<cmath>=\boxed{\textbf{(B)}-\frac{1}{40}}</cmath>

Revision as of 23:17, 3 February 2016

Solution

\[\text{rem}\left(\frac{3}{8},-\frac{2}{5}\right)\] \[=\frac{3}{8}-\left(-\frac{2}{5}\right)\lfloor\frac{\frac{3}{8}}{-\frac{2}{5}}\rfloor\] \[=\frac{3}{8}+\left(\frac{2}{5}\right)\lfloor -\frac{15}{16}\rfloor\] \[=\frac{3}{8}+\left(\frac{2}{5}\right)\left(-1\right)\] \[=\frac{3}{8}-\frac{2}{5}\] \[=\boxed{\textbf{(B)}-\frac{1}{40}}\]